Verify Solution of Matrix Row Reduction Problem

Click For Summary
The matrix was row reduced to its reduced row echelon form (RREF), resulting in the matrix [1 0 -1 -2; 0 1 2 3; 0 0 0 0]. The vector (0,1,2,3) is identified as a linear combination of the other two vectors and is therefore excluded from the basis. The remaining vectors {(1,0,-1,-2), (0,1,2,3)} form a basis for the row space of the matrix. The solution and reasoning provided are confirmed to be correct.
temaire
Messages
275
Reaction score
0

Homework Statement


http://img708.imageshack.us/img708/7309/10324398.jpg

The Attempt at a Solution


First I row reduced the matrix into the RREF and got

<br /> \left[ \begin{array}{cccc} 1 &amp; 0 &amp; -1 &amp; -2 \\ 0 &amp; 1 &amp; 2 &amp; 3 \\ 0 &amp; 0 &amp; 0 &amp; 0 \end{array} \right]<br />

(0,1,2,3) is a linear combinatin of (1,2,3,4) and (2,6,10,14) so you drop it. Therefore {(1,0,-1,-2), (0,1,2,3)} is a basis of row(A).

Am I correct?
 
Last edited by a moderator:
Physics news on Phys.org
yes.
 
Question: A clock's minute hand has length 4 and its hour hand has length 3. What is the distance between the tips at the moment when it is increasing most rapidly?(Putnam Exam Question) Answer: Making assumption that both the hands moves at constant angular velocities, the answer is ## \sqrt{7} .## But don't you think this assumption is somewhat doubtful and wrong?

Similar threads

Replies
1
Views
1K
  • · Replies 2 ·
Replies
2
Views
2K
  • · Replies 2 ·
Replies
2
Views
2K
  • · Replies 6 ·
Replies
6
Views
1K
Replies
15
Views
2K
  • · Replies 5 ·
Replies
5
Views
1K
Replies
6
Views
2K
  • · Replies 9 ·
Replies
9
Views
2K
  • · Replies 8 ·
Replies
8
Views
3K
  • · Replies 3 ·
Replies
3
Views
1K